Who can answer this question? ( I will ask questions every Thursday!!

Who Can Answer This Question? ( I Will Ask Questions Every Thursday!!

Answers

Answer 1

Answer:

32%

Step-by-step explanation:

if it is wrong sorry

Hope it helped have a good weekend;)

Answer 2

Answer:

14 %

Step-by-step explanation:

3 divided by 100 is 0.03 = 3%

14 divided by 100 is 0.14 = 14%

8 divided by 100 is 0.08 = 8%


Related Questions

HELP ME PLEASE!
I like dont understand

Answers

Answer:

5) [tex]8k^{5}[/tex]

7)[tex]6y^{2}x[/tex]

9)[tex]\frac{12}{ab}[/tex]

Step-by-step explanation:

I think the answer is 8k 5

A landscaper mowed 2 1/4 yards in 1 1/2 hours. How much did she mow in 1 hour? How many hours does it take to mow one yard?

Answers

Answer:

1.5 or 1 1/2 of a lawn in one hour. about one hour to mow one yard.

Step-by-step explanation:

It would be 1.5 or 1 1/2.

find the gcf of 30 and 66

Answers

Answer:

6 is the GCF of 30 and 66

Step-by-step explanation:

The GCF of two non-zero integers, x(30) and y(66), is the greatest positive integer m(6) that divides both x(30) and y(66) without an remainder.

Help!*

Mee!** please **!!!

Answers

Answer:

-6.3+d> 4 1/2

Step-by-step explanation:

She starts 6.3 miles west of her home. This is denoted by the (-). The store is 4.5 miles east of her home. East would be denoted by a (+). never be exactly 4.5 miles from her home. This would be denoted by a (>). Therefore the answer is: -6.3+d>4 1/2

The store is more than 4.5 miles east of her home meaning that it would never be exactly 4.5 miles from her home. This would be denoted by a (>). Therefore the answer is: -6.3+d>4 1/2

Two positive improper fractions are multiplied.Is the product sometimes,always,or never less than 1?Explain.
Someone help me please and thank you:)

Answers

Answer:

never

Step-by-step explanation:

Positive fractions will be more then one.

Never positive fractions

USE PICTURE TO ANSWER QUESTION

Answers

Answer:

k = 24

y = 24x

Step-by-step explanation:

72/3 = 24

y = 24x

72 = 24(3)

72 = 72

Hope this helps!

K=24
Y=24x

I hope this helped

help help pls help quick pls help help

Answers

Answer:

d) 1 1/9

Step-by-step explanation:

hope this helps!!

Answer:

is d trust me

An item is regularly priced at $65. It is now priced at a discount of 40% off the regular price. What is the price now?

Answers

39 dollars

Reason for is because our original price is 65 now take away 40% of that which is 26 , 65-26=39

Hellppp meeeeeee PLEASEEEEE :(

Answers

Answer: 29.5 inches tall.

Step-by-step explanation:

If Eva was 1.5 inches taller than her original height, she would be half of Maria’s height. So, we divide Maria’s height by two to find the height Eva would be if she was 1.5 inches taller.

62 / 2 = 31 inches

Next, to find Eva’s real height, we subtract 1.5 inches from the 31 inches. This is because Eva would only be half Maria’s height IF she was 1.5 inches taller, but she is not, so therefore we can deduct it.

31 - 1.5 = 29.5 inches

Therefore, Eva’s height is 29.5 inches tall. We can prove this, as well.

29.5 + 1.5 = 31 inches

31 * 2 = 62 inches

Therefore, Eva is 29.5 inches tall.

Sorry I don’t know.

Evalute 8a -1+0.5b when a =1/4 and b= 10 how dose this work i need help..

Answers

8a -1+0.5b

Replace a nd b with the given values:

8(1/4) -1 + 0.5(10)

8 x 1/4 = 2

0.5 x 10 = 5

Now you have:

2 -1 + 5

2-1 = 1 +5 = 6

Answer is 6

Answer:

6

Step-by-step explanation:

Substitute the value of the variable into the expression and simplify.

FREE BRAINLYEST :))))))))))))))))))))))))))))))))))

Answers

That’s so nice of you! <3 I hope you have an amazing day

<333333333333333333333333333

DUE AT 8 MT!!! HELP ASAP!!!
Define a variable, set up an equation, then solve.
The sum of two numbers is 91. The smaller number is thirteen less than the larger number. Find both numbers.
x - large number
y - small number

Answers

Answer:

x=52 and y=39

Step-by-step explanation:

52 + 39 = 91

52 - 13 = 39

What you would do is make the equation:

91=X+(X-13)

Then, you would solve for X and once you find X, you would subtract 13 from it.

The answers should be 52 and 39.

To rent a certain meeting room, a college charges a reservation fee of $19 and an additional fee of $4 per hour. The chemistry club wants to spend at most $47 on renting the room. What are the possible numbers of hours the chemistry club could rent the meeting room?

Answers

Answer:

The possible number of hours the chemistry club can rent the meeting room is 7 hours.

Step-by-step explanation:

h=hours

19+4h=47

Subtract 19 from both sides of the equation.

4h=28

Divide 4 from both sides of the equation.

h=7

The possible number of hours the chemistry club can rent the meeting room is 7 hours.

Hope this helps :)

Answer:

7 hours

Step-by-step explanation:

There is $47 total dollars. The reservation fee is $19 dollars. Subtract the two:

47-19= 28 dollars

Now the per hour fee is 4 dollars an hour. And you have 28 hours. So divide the two to get maximum hours:

28/4=7 hours

HELPPPPPPPPPPPPPP MEEEEEEEEEEEEEE

Answers

Answer:

Question A: A

Question B: C

Question C: B

Step-by-step explanation:

c, is c. they have already concluded data. the older the age, the shorter the stay. that’s the end conclusion.

PLEASE HELLP 100 POINTS ARE AVAILABALE

Answers

Answer:

A

Step-by-step explanation:

4 (0.5x + 2.5y - 0.7x - 1.3y + 4)  

Use commutative property with like terms 4( 0.5 x - 0.7x + 2.5y - 1.3 y + 4)

Combine like terms 4 (-0.2x + 1.2y + 4)

Use distributive property -0.8x + 4.2y + 16

solve the proportion

Answers

Answer:

3. m= -15

4. x= 9

Step-by-step explanation:

3. Cross multiply: -6m=90

Solve: m= -15

4. Cross multiply: 7x+7=10x-20

Solve: x=9

Step-by-step explanation:

Here,

-m=10*9/6

or,-m=90/60

or,-m=15

or,-m=-(-15)

and, m=-15

2

Here,

(x+1)*7=(x-2)*10

or,7x+7=10x-20

or,7+20=3x

or,x=27/3

therefore, x=9ans

-3.5(10x-2)=-168
solve equation, show all work. is it no solution, one solution or infinite solutions?

Answers

-3.5(10x-2)=-168
/-3.5. /-3.5
=48


10x-2=48
+2 +2

10x=50
/10 /10


x=5
The answer is one solution

help me pls
(if u don't help and just want points i will report)

Answers

Answer:  [tex]20-\frac{1}{4}x+\frac{4}{3}y[/tex]

================================================

Work Shown:

[tex]P = 2L + 2W\\\\P = 2\left(10-x+\frac{4}{9}y\right) + 2\left(\frac{7}{8}x+\frac{2}{9}y\right)\\\\P = 2\left(10\right)-2x+2\left(\frac{4}{9}y\right) + 2\left(\frac{7}{8}x\right)+2\left(\frac{2}{9}y\right)\\\\[/tex]

[tex]P = 20-2x+\frac{8}{9}y + \frac{14}{8}x+\frac{4}{9}y\\\\P = 20-2x+\frac{8}{9}y + \frac{7}{4}x+\frac{4}{9}y\\\\P = 20+\left(-2x + \frac{7}{4}x\right)+\left(\frac{8}{9}y+\frac{4}{9}y\right)\\\\[/tex]

[tex]P = 20+\left(-\frac{8}{4}x + \frac{7}{4}x\right)+\left(\frac{8}{9}y+\frac{4}{9}y\right)\\\\P = 20+\left(\frac{-8+7}{4}x\right)+\left(\frac{8+4}{9}y\right)\\\\P = 20-\frac{1}{4}x+\frac{12}{9}y\\\\P = 20-\frac{1}{4}x+\frac{4}{3}y\\\\[/tex]

The answer is 20-1/4x+4/3y

help pls and letter d is 4 1/2

Answers

The answer is B

12 divided by 3/2 is the same as 12 x 3/2

Do this to 8 to get the answer B

Answer: The answer is B.

Step-by-step explanation:

To figure out, you must first turn the mixed fraction (3 1/2) into an improper fraction (7/2). We can do so by:

3 * 2 = 6

6 + 1 = 7 or 7/2

Next, you follow the rules of KCF: Keep, Change, Flip. Keep the first fraction, Change the sign to its inverse operation, and Flip the second fraction.

Our original equation is 7/2 /(divided by) 3/2 = ?

1. Keep the first fraction

7/2 / 3/2 = 7/2 / 32

2. Change the sign to its inverse operation

7/2 / 3/2 = 7/2 * 3/2

3. Flip the second fraction

7/2 * 3/2 = 7/2 * 2/3

Now, solve.

7/2 * 2/3

7 * 2 / 2 * 3 = 14/6

To make this a proper mixed number, first simplify.

14/6 / 2 = 7/3

Now, figure out how many wholes are in 7/3.

7/3 / 3 = 2 1/3

This is because 2 times 3 is six, and 7 minus 6 equals 1, or 1/3.

Therefore, the answer is 7/3 OR 2 1/3 OR B

James bought a pound of chicken for $2.99, a pound of cheese for$5.99, and a bag of potatoes for $3.99. Find his total at checkout if the sales tax is 7%

Answers

Answer:

Its about 0.90$

Step-by-step explanation:

Hope I get it right. Please mark brainliest if I do thanks! :)

Answer:

13.88

Step-by-step explanation:

Help me out! I will give brainliest! Pleaseeeeee

Answers

A will be your answer

Can someone please help me. I will give brainliest

Answers

Answer:

A

Step-by-step explanation:

Drag each angle measure to the correct location on the image. Each angle measure can be used more than once.

In the figure, two parallel lines are cut by two other parallel lines. The measures of one of the angles is labeled. Find the measures of the other angles and label them.

137° 52° 128° 38°

*then it’s the picture*

Answers

Answer:

In front of the 52, it'll be 52°, that's all I know.

Vertical angles are always congruent. Since the lines are parallel, we can use the Alternate Interior Angle Theorem.

The way I got 128 was by using the fact that the angles would add up to 360 and the Vertical Angle Congruence Theorem. Basically, the formula is x + x + y + y = 360°. If you replace x with 52°, we can deduce that y is equal to 360 - (52 + 52)/2. That gives us 128°.

Please help me my homework is due in 30 mins and I'm stuck on the last question. In this question, I'm working with a recipe where I have to make a recipe for 12 a recipe for 30. "Notice that the original recipe calls for [tex]\frac{1}{4}[/tex] cup of water. If you put [tex]\frac{3}{8}[/tex] cup of water, by how much have you multiplied the original recipe? How many people will your new recipe serve?".

Answers

Answer:

2.5 times, 30 people

Step-by-step explanation:

The recipe will be multiplied 2.5 times (3/8 ÷ 1/4). Therefore, it will serve 30 people (12 * 2.5)

Answer:

30

Step-by-step explanation:

Bro HELPPPPPPPPPPPPPPPPPPPPPPPPP ;-;

Answers

Answer:

27

Step-by-step explanation:

Answer:

$27

Step-by-step explanation:

substitute the value of p (price of the package of henna powder) which has been given as $12 and the value of s (price of the squeeze bottle) which has been stated as $3 into the equation from part b).

since the equation is 3(p-5) + 2s

write 12 instead of p, and 3 instead of s.

then solve the equation.

= 3(12-5) + 2(3)

= 3(7) + 2(3)

= 21 + 6

= 27

fy the sequences of transformations based on whether or not they prove the congruency of the shapes by mapping shape I ont a reflection across the y-axis, followed by a 90° clockwise rotation about the origin, and then a translation left 18 units a 90° clockwise rotation about the origin and then a translation left 18 units a 90° counterclockwise rotation about the origin, followed by a reflection across the y-axis, and then a translation left 18 units a 90° clockwise rotation about the origin, followed by a reflection across the y-axis, and then a translation right 2 units a 180° rotation about the origin, followed by a reflection across the y-axis, and then a 90° clockwise rotation about the origin a reflection across the y-axis, followed by a 90° counterclockwise rotation about the origin, and then a translation right 2 units Maps Shape I onto Shape II Does not Map Shape l onto Shape II​

Answers

Answer:

Step-by-step explanation:

10, 8

-8, -8

Transformation is a method required to resize or change the orientation of a given shape or figure. The required answer is:

i. Statements b and d would map Shape l onto Shape II​ii. Statements a, c, e, and f does not map Shape l onto Shape II​

Transformation is a method required to resize or change the orientation of a given shape or figure. The types of transformation are translation, reflection, rotation, and dilation.Translation is a method that requires moving every point in a given shape in the same direction and same unit.

Reflection is a method that involves flipping a given figure about a given reference point or line. Rotation is a method that required turning a given figure at an angle about a reference point. Dilation is a method in which the length of the sides of the figure is either increased or decreased.Thus the answers to the question are listed as follows:

a. a reflection across the y-axis, followed by a 90° clockwise rotation about the origin, and then a translation left 18 units. Does not Map Shape l onto Shape II​b. a 90° clockwise rotation about the origin and then a translation left 18 units Answer: Maps Shape l onto Shape II​c. a 90° counterclockwise rotation about the origin, followed by a reflection across the y-axis, and then a translation left 18 units d. Does not Map Shape l onto Shape II​d. a 90° clockwise rotation about the origin, followed by a reflection across the y-axis, and then a translation right 2 units Answer: Maps Shape l onto Shape II​e. a 180° rotation about the origin, followed by a reflection across the y-axis, and then a 90° clockwise rotation about the originAnswer: Does not Map Shape l onto Shape II​f. a reflection across the y-axis, followed by a 90° counterclockwise rotation about the origin, and then a translation right 2 units. Does not Map Shape l onto Shape II​.

For more clarifications on the reflection of a figure, visit: https://brainly.com/question/20840412

#SPJ1

Help me out and answer correctly and if it's correct I will give brainliest

Answers

I think it is ethier D or C

Answer:

it should be d which is 7.50

Step-by-step explanation:

I know this because the given numbers are close to this answer plus how can it be 4 . If am wrong pls say so

Question 7(Multiple Choice Worth 1 points)

(04.04 MC)

At Priscilla's Purse Warehouse, it takes 2 over 3 of a day to complete 1 over 9 of an order of purses. At this rate, how long will it take to complete the entire order of purses?

3 and 2 over 3 days
5 days
6 days
13 and 1 over 2 days

Answers

I think it would take 5 days

Answer: I believe its 5 days.

Step-by-step explanation: Hope this helps you!!!!

A cell phone company charges a monthly fee of $50 for 300 minutes. Each addition minute costs $0.05. Determine if the function is linear or no linear and explain your reasoning.

Answers

Answer:

linear

Step-by-step explanation:

The graph would be linear since it's .05 every minute, which would mean that every minute it would cost .05 --> adding up minutes. The flat fee at the beginning is trying to trick you up, don't get fooled ;)

It is linear because the y intercept would start at 50 dollars on a graph and then be constant, making a line

Someone pls help me and I will give brainliest of you answer correctly. plssssss

Answers

I believe the answer is D.
The answer is c yyyyyyyyyyyyyyyyyyooooooouuuuur welcome
Other Questions
(onomatopoeia, alliteration, metaphor, simile, oxymoron, personification, hyperbole,Euphemism, Irony, onomatopoeia)1. I'll finish washing dishes faster than you can blink. Which of the following does a secure website use to safeguard transmitted information What is the solution to this equation? Complete the statement:If AB, CD, thena. AB - CDb. AB = CDC. ABCDd. AB - CD- What type of substance is magnesium carbonate? Leah is making cookies for her birthday party she wants to double the recipe and uses 1cup of sugar and 3 1/2 cups of flour.how many cups of sugar and flower does the original recipe call for. Pls help with the second one. Thank you to the person who helped me with the first one. A compound is 2. 00% H by mass, 32. 7% S by mass, and 65. 3% O by mass. What is its empirical formula? The first step is to calculate the mass of each element in a 100-g sample of the compound. We use 100 g because it makes the calculations more straightforward. What is the mass of each element in a 100. 0-g sample of the compound? mass H = g mass S = g mass O = g. Discuss the use of figurative language in the poem I met a thief by Austin Bukenya What products would you expect from the reaction of ammonia and sulfuric acid in aqueous solution the proteins specific sequence is? through: (5, 4), slope = undefinedwrite the slope intercept Check here LOADING... for instructional material to complete this problem.Evaluate the formula z=pppqn when p=3881386, n=1386, p=0.30, and q=1p in the monohybrid crosses performed by mendel, the f1 plants always displayed ____. How many quarters are there in 5 1/2 What is the range of this set of numbers? 8, 4, 5, 1, 6, 2, 6, 9, 6, 3 A. 6 B. 7 C. 5 D. 8 A clothing company makes a profit of $2.3 million. This is $4.1 million more than last year. What was the profit last year? If correct Ill give you brainly (2^4x+4)(2^x+1)60 POINTSBRANLIEST IF RIGHT in what part of the neuron does the action potential typically initiate?